How to Prove the Integral Property for Definite Integrals

Click For Summary
The discussion focuses on proving the integral property for definite integrals, specifically using the symmetry of the sine function around π/2. A suggested substitution is x = π - t, which simplifies the proof and allows for solving the integral. The original poster struggles to understand how to initiate the proof and seeks guidance on the substitution method. The conversation emphasizes the importance of recognizing function symmetry in calculus problems. Understanding these concepts is crucial for mastering definite integrals.
doktorwho
Messages
181
Reaction score
6

Homework Statement


Today i had a test on definite integrals which i failed. The test paper was given to us so we can practise at home and prepare better for the next one. This is the first problem which i need your help in solving::
Test.JPG


Homework Equations


3. The Attempt at a Solution [/B]
As no points were given for a solution of the below integrals without the proof of the integral property above i need to do that first. I had no idea how to start the proof. I figured i need to use some sort of substitution but i fail to see which and why. Could you give me a hint on how to do this? I know i haven't provided any work done by myself but i can't since i can't start. I didn't have a clue calculus was going to be this hard :/.
Thanks
 
Physics news on Phys.org
The key here is that ##sin(x)## is symmetric around ##pi/2##. Hence the substitution ##t=pi/2+x## may be of use. You then see that a term in your new expression should disappear.
 
Incand said:
The key here is that ##sin(x)## is symmetric around ##pi/2##. Hence the substitution ##t=pi/2+x## may be of use. You then see that a term in your new expression should disappear.

I think you probably meant something more like ##x=\pi-t##.
 
  • Like
Likes nuuskur
The substitution ##x = \pi - t ## does indeed do the trick for proving the proposition before solving the given integral.
 
nuuskur said:
The substitution ##x = \pi - t ## does indeed do the trick for proving the proposition before solving the given integral.
Yeah i did it with the substitution you proposed but how did you arrive at it?
 
Question: A clock's minute hand has length 4 and its hour hand has length 3. What is the distance between the tips at the moment when it is increasing most rapidly?(Putnam Exam Question) Answer: Making assumption that both the hands moves at constant angular velocities, the answer is ## \sqrt{7} .## But don't you think this assumption is somewhat doubtful and wrong?

Similar threads

  • · Replies 7 ·
Replies
7
Views
1K
Replies
2
Views
2K
Replies
20
Views
2K
  • · Replies 4 ·
Replies
4
Views
2K
  • · Replies 6 ·
Replies
6
Views
2K
  • · Replies 22 ·
Replies
22
Views
2K
  • · Replies 44 ·
2
Replies
44
Views
6K
  • · Replies 2 ·
Replies
2
Views
3K
  • · Replies 11 ·
Replies
11
Views
1K
Replies
14
Views
2K